Every year, Shannon and her sister attend 'The Nutcracker' at the Arcadia Ballet. Last year, orchestra seating cost $50 per ticket. This year, each ticket was 14% more expensive. What was the cost of each ticket this year?

Answers

Answer 1

Answer:

$57

Step-by-step explanation:

14% of $50 is:

.14 × 50

= 7

This means the ticket was $7 more.

The price was $50 + $7

Each ticket cost $57.


Related Questions

Sophie invested $92,000 in an account paying an interest rate of 6 1/8% compounded

continuously. Damian invested $92,000 in an account paying an interest rate of 6 5/8%

compounded monthly. After 14 years, how much more money would Damian have in

his account than Sophie, to the nearest dollar?

Answers

Answer:

Step-by-step explanation:

To solve this problem, we need to use the formula for compound interest:

A = P*e^(rt)

where A is the final amount, P is the principal (initial investment), e is the base of the natural logarithm (approximately 2.71828), r is the interest rate (expressed as a decimal), and t is the time (in years).

For Sophie's account, we have:

P = $92,000

r = 6 1/8% = 0.06125 (as a decimal)

t = 14 years

A = 92000*e^(0.06125*14)

A = $219,499.70 (rounded to the nearest cent)

For Damian's account, we have:

P = $92,000

r = 6 5/8% = 0.06625/12 = 0.005521 (as a monthly decimal rate)

t = 14*12 = 168 months

A = 92000*(1+0.005521)^168

A = $288,947.46 (rounded to the nearest cent)

Now we can subtract Sophie's final amount from Damian's final amount to find the difference:

Difference = $288,947.46 - $219,499.70

Difference = $69,447.76

Therefore, Damian would have about $69,448 more in his account than Sophie, to the nearest dollar.

Determine the force in each member of the truss with the method of joints and state if the

members are in tension (T) or compression (C). Set d = 1 m and P = 10 kN. (Hint: Look for zero-

force members to simplify the calculations)

Answers

if the members are in tension or compression. Identify all zero force members: Likewise, we can find the reaction force at A by taking minutes about point A: RA x 8m - 5kN x 8m - 5kN x 8m = 0 RA = 5kN

To begin with, we really want to find the reaction forces at An and G.

We can do this by taking minutes about point G.

We realize that the amount of minutes at any point is zero when the framework is in equilibrium.

Consequently, we can compose: 5kN x 8m - RA x 10m = 0 RA = 4kN

Likewise, we can find the reaction force at A by taking minutes about point A: RA x 8m - 5kN x 8m - 5kN x 8m = 0 RA = 5kN

Since we have two distinct qualities for RA, we can presume that the framework isn't in equilibrium.

This really intends that there should be some outside force following up on the framework.

The two obscure forces are at first thought to be ductile (for example pulling away from the joint). In the event that this underlying supposition is mistaken, the registered upsides of the pivotal forces will be negative, meaning pressure.

to know more about forces click here:

https://brainly.com/question/30284058

#SPJ4

the complete question is:

Question l Find the forces in members HE; FH, FE; and FC of the truss as shown in Figure Q1. State if the members are in tension or compression. Identify all zero force members: (10 marks) 8 m 5 KN 8 m 8 m 5 KN 8 m 10 m Figure Q1.

find the values of a and b such that
x^2- +5=(x-a)^2+b

Answers

The value οf a = 1/2  and b = 19/4 in the equatiοn x² - x + 5 = (x-a)² + b.

What dο yοu mean by algebra?

The part οf mathematics in which letters and οther general symbοls are used tο represent numbers and quantities in fοrmulae and equatiοn is called algebra.

x² - x + 5 = (x-a)² + b

x² - x + 5 = x² + a² - 2xa + b

-x + 5 = -2xa + a² + b

By matching cοrrespοnding terms,

2a = 1          and        a²+ b = 5

a= 1/2          and        a²+ b = 5

Substituting value οf "a"

(1/2)² + b = 5

1/4 + b = 5

b = 5- 1/4

b = 19/4

Thus, a = 1/2  and b = 19/4.

To learn more about algebra, click on the link given below

https://brainly.com/question/22399890

#SPJ1

A photograph of sides 35cm by 22cm is mounted onto a frame of external dimension 45cm by 30cm.Find the area of the border surrounding the photograph

Answers

Solution:

Dimension of photograph is 35cm and 22cm.

And external dimension of photo frame is 45cm and 30cm

So, the area of the border surrounding the photograph=Area of photo frame−Area of photo.

So, The area of the border surrounding the photograph [tex]=45\times30-35\times22[/tex]

[tex]=1350-770=580cm^2[/tex]

Terri pays a monthly cell phone fee of $10. She pays 5 cents for each minute that she talks. If Terri does not make any calls, what would her bill be?

Answers

Answer:

$10

Step-by-step explanation:

We Know

Terri pays a monthly cell phone fee of $10. She pays 5 cents for each minute that she talks.

Let C be the total cost, and x be the number of minutes she talks; we have the equation.

C = 0.05x + 10

If Terri does not make any calls, what would her bill be?

C = 0.05(0) + 10

C = $10

So, her bill will be $10

PLEASE HELP!!! WILL MARK BRANLIEST!!!

Answers

Answer:

The point z = 3+4i is plotted as a blue dot, and the two square roots are plotted as a red dot and a green dot. The magnitudes of z and its square roots are shown by the radii of the circles centered at the origin.

Step-by-step explanation:

qrt(z) = +/- sqrt(r) * [cos(theta/2) + i sin(theta/2)]

where r = |z| = magnitude of z and theta = arg(z) = argument of z.

Calculate the magnitude of z:

|r| = sqrt((3)^2 + (4)^2) = 5

And the argument of z:

theta = arctan(4/3) = 0.93 radians

Now, find the two square roots of z:

sqrt(z) = +/- sqrt(5) * [cos(0.93/2) + i sin(0.93/2)]

= +/- 1.58 * [cos(0.47) + i sin(0.47)]

= +/- 1.58 * [0.89 + i*0.46]

Using a calculator, simplify this expression to:

sqrt(z) = +/- 1.41 + i1.41 or +/- 0.2 + i2.8

Suppose that a phone originally sold for $800 loses 3/5 of its value each year after it is released. After 2 years, how much is the phone worth?A. $800B. $1333C. $128D. $288​

Answers

The phone is worth $128 after 2 years, which is option C.

What is exponential decay ?

Exponential decay is a decrease in a quantity over time where the rate of decay is proportional to the current value. In this case, the value of the phone decreases by 3/5 each year after it is released. This means that the value after one year is 2/5 of the original value, and the value after two years is (2/5) times (2/5) of the original value. Exponential decay is a common phenomenon in many areas of science and mathematics, including radioactive decay, population growth and decay, and financial investments.

Calculating the worth of the phone :

The phone is not worth the same amount after 2 years, as it loses 3/5 of its value each year. We need to calculate its worth after 2 years.

Let's use the formula for exponential decay: [tex]A = A_0(1 - r)^t[/tex], where A is the final amount, [tex]A_0[/tex] is the initial amount, r is the decay rate, and t is the time elapsed.

In this case, the initial amount is $800, the decay rate is 3/5, and the time elapsed is 2 years. Substituting these values into the formula, we get:

[tex]A = 800(1 - 3/5)^2[/tex]

[tex]A = 800(2/5)^2[/tex]

[tex]A = 800(4/25)[/tex]

[tex]A = 128[/tex]

Therefore, the phone is worth $128 after 2 years, which is option C.

To know more about exponential decay visit :

brainly.com/question/2193799

#SPJ1

smart mugs are the next generations of hot drinks dispensers tha om with built in technology to keep drinks at the perfect temperature for hours on end initial cost of a smart mug was aed 896, because of high demand in market the cost increased by 12% find the new price of the mug



PLS QUICK ITS DUE 1 HOUR!!

Answers

With a [tex]12[/tex]% price rise, the smart mug now costs AED [tex]1003.52[/tex].

Price and sell price: what are they?

The sale price is the price an user pays to purchase a thing or a commodity. It is a cost that is higher than the market cost and also includes a portion of the profit. The cost price refers to the price paid by the seller for the item or service.

How would you define price?

Price is the process of figuring out how much a something or service is worth. Price establishes a customer's cost, although it can or cannot be linked to the price a firm pays to manufacture a good or service.

We need to multiply the initial price by [tex]1.12[/tex] which represents a [tex]12[/tex]% increase in decimal form,

New price [tex]=[/tex] Initial price [tex]*[/tex] (1 [tex]+[/tex] Percent increase in decimal form)

New price[tex]= 896 * (1 + 0.12)[/tex]

New price [tex]= 896 * 1.12[/tex]

New price [tex]= 1003.52[/tex]

Therefore, the new price of the smart mug is AED [tex]1003.52[/tex] after a [tex]12[/tex]% increase.

To know more about price visit:

https://brainly.com/question/19091385

#SPJ1

Subtract the given equation

3x-(4x-11)

Answers

Answer:

3x - (4x - 11) = 3x - 4x + 11 = -x + 11

Step-by-step explanation:

one hundred students were asked whether they liked certain candy flavors. it was found that liked cherry, liked coconut, and liked both flavors. what's the probability that a randomly selected student...

Answers

The probability that a randomly selected student likes coconut is 0.6.

Step-by-step explanation:

Given that,

The number of students = 100 The number of students who liked cherry = 30 The number of students who liked coconut = 60 The number of students who liked both flavors = 10

Now we have to find the probability that a randomly selected student likes coconut.

P (student likes coconut) = the number of students who liked coconut / total number of students

= 60/100

= 0.6

Therefore, the probability that a randomly selected student likes coconut is 0.6.

See more about probability at: https://brainly.com/question/13604758

#SPJ11

what is this pls help

Answers

Answer:

x = 45.

Step-by-step explanation:

We know the full angle of this is 180 degrees.

Given: (2x+45) + x = 180

First, collect like terms ( in this case 2x and x, 180 and 45 )

2x + x = 180 - 45

Then calculate:

3x = 135. ( Divide both sides by 3 )

x = 45

g in acid base reactions, the hydrogen ions from the acid and the hydroxide ions from the base neutralize each other. khp has one ionizable hydrogen; this means that one mole of sodium hydroxide neutralizes one mole of khp. from experiment 1, calculate the exact molarity of the sodium hydroxide. (hint: use the mass of khp and do a stoichiometry problem.....) tip: khp is not the chemical formula. khp stands

Answers

In the following question, among the conditions given, the statement is said to be, the exact molarity of the NaOH solution is 0.0960 M.

The question is asking to calculate the exact molarity of the sodium hydroxide from Experiment 1.
KHP stands for potassium hydrogen phthalate, and one mole of sodium hydroxide (NaOH) will neutralize one mole of KHP. To solve the problem, use the mass of KHP and a stoichiometry problem.
First, calculate the number of moles of KHP:
Moles KHP = (Mass KHP (g) / Molar Mass KHP (g/mol))
Then, calculate the moles of NaOH:
Moles NaOH = (Moles KHP * Mole Ratio NaOH/KHP)
Finally, calculate the molarity of NaOH:
Molarity NaOH = (Moles NaOH / Volume NaOH (L))

For more such questions on NaOH

https://brainly.com/question/15973235

#SPJ11

A local winery wants to create better marketing campaigns for its white wines by understanding its customers better. One of the general beliefs has been that higher proportion of women prefer white wine as compared to men. The company has conducted a research study in its local winery on white wine preference. Of a sample of 400 men, 120 preferred white wine and of a sample of 500 women, 170 preferred white wine. Using a 0.05 level of significance, test this claim.INPUT Statistics required for computation170 = Count of events in sample 1500 = sample 1 size120 = Count of events in Sample 2400 = sample 2 size0.05 = level of significance0 = hypothesized differenceOUTPUT Output valuesSample 1 Proportion 34.00%Sample 2 Proportion 30.00%Proportion Difference 4.00%Z α/2 (One-Tail) 1.645Z α/2 (Two-Tail) 1.960Standard Error 0.031Hypothesized Difference 0.000One-Tail (H0: p1 − p2 ≥ 0)Test Statistics (Z-Test) 1.282p-Value 0.900One-Tail (H0: p1 − p2 ≤ 0)Test Statistics (Z-Test) 1.282p-Value 0.100Two-Tail (H0: p1 − p2 = 0)Test Statistics (Z-Test) 1.276p-Value 0.202Group of answer choicesThis is a one-tail test and the data does support the claim that higher proportion of women prefer white wine as compared to men.This is a one-tail test and the data does not support the claim that higher proportion of women prefer white wine as compared to men.This is a two-tail test and the data does support the claim that higher proportion of women prefer white wine as compared to men.This is a two-tail test and the data does not support the claim that higher proportion of women prefer white wine as compared to men.Question 2. Based on the study results presented in the last question, what is the upper bound for the proportion differences between women and men for a 95% confidence interval?(Note: Please enter a value with 4 digits after the decimal point. For example, if you computed an upper boundary of 23.456% or .23456, you would enter it here in decimal notation and round it to four digits, thus entering .2346).

Answers

Answer:

235.65

Step-by-step explanation:

Analyze the proportion below and complete the instructions that follow. Use a model to find the missing value in the proportion. A. 4 B. 5 C. 10 D. 22 Please select the best answer from the choices provided A B C D

Answers

Step-by-step explanation:

The area of a rectangle is 1,872 ft2. The ratio of the length to the width is 9:13. Find the perimeter of the rectangle.

176 ft

You want to make a scale drawing of your bedroom to help arrange your furniture. You decide on a scale of 3 in. = 2 ft. Your bedroom is a 12 ft by 14 ft rectangle. What should the dimensions of your drawing be?

18 in. by 21 in.

If 5/y + 7/x=24 and 12/y + 2/x=24, find the ratio of x to y.

5/7

Simplify the ratio 8ft/12in. Use the conversion 12 in. = 1 ft.

8/1

Analyze the proportion below and complete the instructions that follow.

2x+5/3 = x-5/4

-7

If a+b/2a-b = 5/4 and b/a+9 = 5/9, find the value of b.

30

Analyze the ratio below and complete the instructions that follow.

$30:$6

Simplify the ratio.

5:1

If 14/3 = x/y then 14/x =

3/y

Analyze the diagram below and complete the instructions that follow.

In the diagram, AB:BC is 3:4 and AC = 42. Find BC.

24

Analyze the diagram below and complete the instructions that follow.

If AB:BC is 3:11, solve for x.

9

If a, b, c, and d are four different numbers and the proportion a/b = c/d is true, which of the following is false?

b/a = c/d

Analyze the diagram below and complete the instructions that follow.

Find the ratio of the width to the length of the rectangle, then simplify the ratio. Use the conversion 100 cm = 1 m.

3/4

Simplify the ratio 3 gal./24 qt. Use the conversion 4 qt = 1 gal.

1/2

The area of a rectangle is 4,320 ft2. The ratio of the length to the width is 6:5. Find the length of the rectangle.

72 ft

Analyze the diagram below and complete the instructions that follow.

Given that CB/CA = DE/DF, find BA.

10.5

Analyze the proportion below and complete the instructions that follow.

2/3 = 8/x

3, 8

Analyze the diagram below and complete the instructions that follow.

Are the polygons shown here similar? Justify your answer. The images are not drawn to scale.

Yes, PQR ~TSV with a scale factor of 1:√3

All __________ are similar.

squares

Analyze the diagram below and complete the instructions that follow.

Determine which 2 triangles are similar to each other. The images are not drawn to scale.

GHI ~ JKL

Analyze the diagram below and complete the instructions that follow.

Pentagon PQRST ~ pentagon XYZVW. Find the value of b. The images are not drawn to scale.

3

Analyze the diagram below and complete the instructions that follow.

If ABC ~ XYZ, find XY. The images are not drawn to scale.

24

ABC is a right triangle. The legs of ABC are 9 ft and 12 ft. The shortest side of XYZ is 13.5 ft, and ABC ~ XYZ How long is the hypotenuse of XYZ?

22.5 ft

Question 9(Multiple Choice Worth 2 points)
(Irrational Numbers LC)
Describe in words where √63 would be plotted on a number line.
O Between 3 and 4, but closer to 3
O Between 3 and 4, but closer to 4
O Between 2 and 3, but closer to 2
O Between 2 and 3, but closer to 3

Answers

It would be plotted Between 3 and 4, but closer to 4

Answer:

Between 3 and 4, but closer to 4

10 is 30% of what number?
Solution:
Part:
whole
percent:

Answers

The number 10 is 30% of the following number: 33.33.

What is a percentage?

In Mathematics, a percentage can be defined as any number that is expressed as a fraction of hundred (100). This ultimately implies that, a percentage indicates the hundredth parts of any given number.

How to calculate the percentage of a number?

In order to determine the whole number which 10 is 30%, we would apply the following mathematical expression (formula);

Quantity = percent × number

Substituting the given parameters into the percentage and quantity formula, we have the following;

10 = 30/100 × number

10 = 0.3  × number

Number = 10/0.3

Number = 33.33.

Read more on percentage here: brainly.com/question/28009735

#SPJ1

Help due soon !!!!!!!!!

Answers

An expression for the length of the rectangle in terms of A is [tex]$\boxed{L=x+5}$[/tex]

How to find the expression?

We are given that the area of a rectangle is [tex]$A=x^2+x-15$[/tex], and we want to find an expression for the length of the rectangle in terms of A.

Recall that the area of a rectangle is given by the formula: [tex]$A=L\cdot W$[/tex], where L is the length and W is the width. We can use this formula to write L in terms of A and W as [tex]$L=\frac{A}{W}$[/tex].

We know that the rectangle has a length and a width, so we need to find an expression for the width W in terms of A. We can rearrange the given formula for A to solve for W:

[tex]&& \text{(substitute }L=x+5\text{)}[/tex]

[tex]W&=\frac{x^2+x-15}{x+5} && \text{(divide both sides by }x+5\text{)}[/tex]

Now that we have an expression for W in terms of A, we can substitute it into our expression for L to get:

[tex]L&=\frac{A}{W}[/tex]

[tex]&=\frac{x^2+x-15}{\frac{x^2+x-15}{x+5}} && \text{(substitute the expression we found for }W\text{)}\&=x+5[/tex]

Therefore, an expression for the length of the rectangle in terms of A is [tex]$\boxed{L=x+5}$[/tex]

To know more about rectangle visit:

https://brainly.com/question/29123947

#SPJ1

I need help with this

Answers

sum area = -3x - 6y + 12 and product area = -36x - 72y.

what is rectangle?

A rectangle is a geometric shape that is defined as a four-sided flat shape with four right angles (90-degree angles) and opposite sides that are parallel and equal in length.

The area of a rectangle is given by the product of its length and width. Assuming that the length of the rectangle is given by -3x - 6y and its width is 12, we can express the area in terms of a sum and a product as follows:

Sum:

Area = length x width

Area = (-3x - 6y) + 12

Area = -3x - 6y + 12

Product:

Area = length x width

Area = (-3x - 6y) x 12

Area = -36x - 72y

Note that the product expression is not equal to the sum expression. This is because we used different assumptions for the length of the rectangle in each case.

Therefore, sum area = -3x - 6y + 12 and product area = -36x - 72y.

To learn more about rectange from the. given link:

https://brainly.com/question/29123947

#SPJ1

The store sells a television for $1000. customers can choose to receive 10% discount and pay it off at a simple interest rate of 4% or they can choose to pay the full price and pay it off in 3 years with no interest. which option is better

Answers

Option 1 with the discount and 4% simple interest has a total cost of $972, while Option 2 with no discount and no interest has a total cost of $1000. Option 1 is the better choice as it has a lower total cost.

What is simple interest?

Simple interest is a type of interest that is calculated on the original principal amount of a loan or investment. It is a fixed percentage of the principal amount that is paid by the borrower or earned by the lender over a specific period of time.

According to question:

To compare the two options, we need to calculate the total cost of each option and compare them.

Option 1: 10% discount and pay off with 4% simple interest

The discount reduces the price of the television to $1000 x 0.9 = $900. If the customer chooses to pay it off at 4% simple interest, the total cost would be:

Total cost = $900 + ($900 x 0.04 x 3) = $972

Option 2: Full price and pay off in 3 years with no interest

The total cost of this option would be simply the full price of $1000 paid over 3 years, so:

Total cost = $1000 / 3 = $333.33 per year x 3 years = $1000

Comparing the two options, we see that Option 1 with the discount and 4% simple interest has a total cost of $972, while Option 2 with no discount and no interest has a total cost of $1000. Therefore, Option 1 is the better choice as it has a lower total cost.

To know more about simple interest visit:

https://brainly.com/question/29210957

#SPJ1

The complete question is: The store sells a television for $1000. customers can choose to receive 10% discount and pay it off at a simple interest rate of 4% or they can choose to pay the full price and pay it off in 3 years with no interest. which option is better?

Option 1 with the discount and 4% simple interest.

Option 2 with no discount and no interest.

what are the roots of 2x^2+10x+9=2x

Answers

The roots of the equation 2x² + 10x + 9 = 2x does not exist i.e no real roots

Calculating the roots of the equation

To find the roots of the given quadratic equation 2x² + 10x + 9 = 2x, we can start by rearranging the equation to the standard form of a quadratic equation

2x² + 10x + 9 - 2x = 0

Simplifying the left-hand side, we get:

2x² + 8x + 9 = 0

Now, we can use the quadratic formula to find the roots of the equation:

x = (-b ± √(b² - 4ac)) / 2a

where a = 2, b = 8, and c = 9.

Substituting these values into the formula, we get:

x = (-8 ± √(8² - 4(2)(9))) / 2(2)

Simplifying the expression under the square root, we get:

x = (-8 ± √-8) / 4

The square root of -8 is not a real number

So, the equation has no real root

Read more about quadratic equation at

https://brainly.com/question/24334139

#SPJ1

I don’t know what I’m doing

Answers

Answer:

29.12 square cm

Step-by-step explanation:

Area of equilateral triangle:

     Side = a = 8.2 cm

    [tex]\boxed{\bf Area \ of \ equilateral \ triangle = \dfrac{\sqrt{3}}{4}a^2}[/tex]

                                                       [tex]\sf = \dfrac{\sqrt{3}}{4}*8.2*8.2\\\\= \sqrt{3}*4.1 * 4.1\\\\= 1.732 * 4.1 *4.1\\\\= 29.12 \ cm^2[/tex]

why can't we use mean when a data set has one or two values that are much higher than all of the others

Answers

The reason we can't use the mean when a data set has one or two values that are much higher than all of the others is that it skews the average, making it not representative of the rest of the data.

What is the mean?

The mean is a numerical measure of the central tendency of a data set. It is calculated by dividing the sum of all the values in a data set by the number of data points.

A data set is a collection of observations or measurements that are analyzed to obtain information. It can be represented graphically, in tabular form, or in any other format. The data set may be a sample or the entire population.

If a data set has one or two extremely high or low values, it can significantly impact the mean. These values are known as outliers. The outliers can cause the mean to be higher or lower than the actual middle value of the data.

Hence, in such cases, the median is a better choice for finding the central tendency of the data. The median is the middle value of the data set, and it is less affected by outliers than the mean. The mode, which is the value that occurs most frequently in the data set, is also a measure of central tendency that is less sensitive to outliers than the mean.

Learn more about mean here:

https://brainly.com/question/31101410

#SPJ11

Jerry, Jack and Sophie are all hoping to save​ money! Jerry thinks saving money in a shoe box in his closet every month is a good idea. He decides to start with​ $125, and then save​ $50 each month. Jack was given​ $3520 from his​ Grandma, and decides to put the money
into an account that has a​ 6.5% interest rate that is compounded annually. Sophie has earned​ $3500 working at the movie theater decides to put her money in the bank in an account that has a​ 7.05% interest rate that is compounded continuously
Part 1​: Describe the type of equation that models​ Jerry’s situation. Create that equation of​ Jerry’s situation. Using the equation you​ created, how much money will be in​ Jerry’s account after 3​ years? 10​ years?
Think​: What do I know and what does it​ mean? What plan am I going to​ try?

PLEASE HELP!!!!!

Answers

Jerry will have $1825 in his account after 3 years and Jerry will have $6125 in his account after 10 year when compounded.

What is simple interest?

Simple interest is computed just using the principle, which is the initial sum borrowed or put into an investment. The interest rate is constant throughout time and solely applies to the principal sum. Short-term loans or investments frequently employ simple interest.

The given situation can be modeled as a linear equation given by:

y = mx + c

For Jerry we have:

y = 50x + 125

For 3 years = 36 months we can substitute x = 36:

y = 50(36) + 125

y = 1825

For x = 10:

y = 50(120) + 125

y = 6125

Hence, Jerry will have $1825 in his account after 3 years and Jerry will have $6125 in his account after 10 year.

Learn more about simple interest here:

https://brainly.com/question/30824126

#SPJ1

PLEASE HELP ME QUICKLY!

Answers

Step-by-step explanation:

it would mean that she made 53 batches of soap and 4 batches of lotion.

now, is it a solution ?

then both inequalities must be true with these values.

5×53 + 15×4 <= 325

265 + 60 <= 325

325 <= 325 correct

20×53 + 35×4 <= 1200

remember, 1 hour = 60 minutes.

1060 + 140 <= 1200

1200 <= 1200 correct

so, (53, 4) is the intersection point of both limit lines. and it is as such an extreme point and optimum.

For the value of the sum, enter an expression that gives the exact value, rather than entering an approximation.
A.-12 + 4 – 4/3 +4/9 – 4/27 +4/81 - … = -12/(1+1/3)
B.∑ (1/3)^n = 6*1/3^6(1/3^11-1)/(1/3-1)

Answers

a. The exact value of the sum of -12 + 4 – 4/3 +4/9 – 4/27 +4/81 - … = -12/(1+1/3)  is 12/7.

b.The exact value of the sum of∑ (1/3)ⁿ = 6*1/3⁶(1/3¹¹-1)/(1/3-1)  is 3/2.

A.-12 + 4 – 4/3 +4/9 – 4/27 +4/81 - …

This is an infinite geometric series with first term a = -12 and common ratio r = 4/(-3). The sum of an infinite geometric series is given by:

S = a / (1 - r)

Substituting the values of a and r, we get:

S = (-12) / [1 - (4/(-3))]

Simplify the denominator by multiplying both numerator and denominator by (-3):

S = (-12) / [-3 - 4]

S = (-12) / (-7)

S = 12/7

Therefore, the exact value of the sum is 12/7.

B. 6*1/3⁶(1/3¹¹-1)/(1/3-1)

This is a geometric series with first term a = 1 and common ratio r = 1/3. The sum of a geometric series with n terms is given by:

S = a (1 - rⁿ) / (1 - r)

As n approaches infinity, rⁿ approaches zero and the sum converges to:

S = a / (1 - r)

Substituting the values of a and r, we get:

S = 1 / (1 - 1/3)

S = 3/2

Therefore, an expression that gives the exact value of the sum is 3/2.

Learn  more about value at https://brainly.com/question/1446179

#SPJ11

There is a 0.99962 probability that a randomly selected 28​-year-old female lives through the year. An insurance company wants to offer her a​ one-year policy with a death benefit of ​$500,000. How much should the company charge for this policy if it wants an expected return of ​$400 from all similar​ policies?

Answers

In order to expect a return on $400 from across all policies of a similar nature, the insurance firm should charge the policy for about $501.88.

How then do we return a value?

Return[expr] leaves control structures that are present during a function's definition and returns the value expression for the entire function. Even if it comes inside other functions, yield takes effect as quickly as it is evaluated. Functions like Scan can use Return inside of them.

Since p is the chance that the 28-year-old woman survives the year and is given as 0.99962, we can enter this number into the equation for n as follows: n = 400(0.99962)/500,400 n 0.799

In light of this, the insurance provider should impose a premium of: Premium = 400/n

$501.88 is the premium ($Premium = 400/0.799)

To know more about return visit:

https://brainly.com/question/28562900

#SPJ1

find the percent of the discount: a $30 board game on sale for 21​

Answers

well, we know the discount is just 30 - 21 = 9, so hmm if we take 30(origin amount) to be the 100%, what's 9 off of it in percentage?

[tex]\begin{array}{ccll} Amount&\%\\ \cline{1-2} 30 & 100\\ 9& x \end{array} \implies \cfrac{30}{9}~~=~~\cfrac{100}{x} \\\\\\ 30x=900\implies x=\cfrac{900}{30}\implies x=30[/tex]

Halla los números desconocidos de estas operaciones
A)872+. +173=2000
B)9180:. =102
C). -99=706
Con los mismos números y las mismas operaciones podemos obtener diferentes resultados,coloca los paréntesis de manera que se obtengan los resultados indicados. A)3+5x7-2=40
B)3+5×7-2=54
C)3+5×7-2=28
ES PARA HOY PORFAVOR☹,PUEDEN HACER EN UNA HOJA O ESCRIBIR ASI PERO EXPLIQUEN BIEN!!!!!!AYUDA SI NO SABEN NO RESPONDAD

Answers

In equation A the missing number is 955, In equation B the missing number is 90 and In equation C the missing number is 805.

A) To find the missing number in the equation 872 + ? + 173 = 2000, we need to subtract 872 and 173 from 2000, which gives us:

2000 - 872 - 173 = 955

Therefore, the missing number is 955.

B) To find the missing number in the equation 9180 ÷ ? = 102, we need to divide 9180 by 102, which gives us:

9180 ÷ 102 = 90

Therefore, the missing number is 90.

C) To find the missing number in the equation ? - 99 = 706, we need to add 99 to 706, which gives us:

706 + 99 = 805

Therefore, the missing number is 805.

To obtain the indicated results with the same numbers and operations, we need to use parentheses to change the order of operations.

A) 3 + (5x7) - 2 = 40

B) (3 + 5) × 7 - 2 = 54

C) 3 + (5 × (7-2)) = 28

Equations are used extensively in various fields of science, engineering, economics, and finance, to name a few. It is formed by placing an equal sign between the two expressions. Equations are used to solve problems and find unknown values.

An equation can contain variables, constants, and mathematical operations such as addition, subtraction, multiplication, and division. The variables in an equation represent unknown values that need to be found, while the constants are known values that are already given. Solving an equation involves manipulating the expressions on both sides of the equal sign using mathematical operations to isolate the variable on one side and constants on the other. The final solution obtained is the value of the variable that satisfies the equation..

To learn more about Equation visit here:

brainly.com/question/29538993

#SPJ4

Complete Question: -

Find unknown numbers of these operations

A ) 872 +. + 173 = 2000

B ) 9180:. = 102

C ). -99 = 706

With the same numbers and the same operations we can obtain different results, place the parentheses so that the indicated results are obtained.

A ) 3 + 5 x 7-2 = 40

B ) 3 + 5 × 7-2 = 54

C ) 3 + 5 × 7-2 = 28

IT'S FOR TODAY PLEASE ☹, CAN DO IN A LEAF OR WRITE ASI BUT EXPLAIN WELL!!!!!!HELP IF THEY DON'T KNOW NO RESPOND

Given the triangle, find the length of X. Give your answer in simpliest radical form.

Answers

Answer:

x = 4[tex]\sqrt{2}[/tex]

Step-by-step explanation:

using the cosine ratio in the lower right triangle and the exact value

cos45° = [tex]\frac{1}{\sqrt{2} } }[/tex] , then

cos45° = [tex]\frac{adjacent}{hypotenuse}[/tex] = [tex]\frac{4}{x}[/tex] = [tex]\frac{1}{\sqrt{2} }[/tex]  ( cross- multiply )

x = 4[tex]\sqrt{2}[/tex]

Mrs banks wants to make 44 quarts of jelly with 70 pounds of fruit if each gallon of jelly costs 6. 5 pounds of fruit will she of enough fruit and will there be extra

Answers

Mrs. Banks has enough fruit to make the 44 qt of jelly she wants, but she will have 4 lb of leftover fruit.

Here we have to use the arithmetic operations. First, we need to convert the total quantity of jelly to gallons since we have the amount of fruit needed per gallon. One gallon is equal to 4 quarts, so 44 quarts is equal to 11 gallons.

Next, we can calculate how much fruit is needed for 11 gallons of jelly by multiplying the amount of fruit needed per gallon by the number of gallons

11 gallons x 6 lb of fruit per gallon = 66 lb of fruit needed

Since Mrs. Banks only has 70 lb of fruit, she has enough to make the 44 qt of jelly she wants, but she will have 4 lb of leftover fruit:

70 lb of fruit - 66 lb of fruit needed = 4 lb of leftover fruit

Learn more about Arithmetic operations here

brainly.com/question/30553381

#SPJ4

The given question is incomplete, the complete question is:

Mrs. Banks wants to make 44 qt of jelly with 70 lb of fruit. If each gallon of jelly needs 6 lb of fruit, will

she have enough fruit? How much leftover fruit does she have, or how much extra fruit is needed?

Other Questions
cognitive psychotherapy is based on the assumption that psychological problems are essentially caused by? the products of protein and carbohydrate digestion are absorbed into the __________; while the products of lipid digestion are absorbed into the _________. I need some help with this at the normal resting membrane potential of a typical neuron, its sodium-potassium exchange pump transports d) H2C2O4 is a stronger acid than HC2O4 an unknown mineral sample scratches fluorite but cannot scratch apatite. what is the approximate hardness of this mineral? Sodium iodine has a pysical life of 8 days and a biological half-life of 24 days. determine its effective half-time When obtaining information on products and services from websites, discussion boards, and blogs, it is important NOT to assume that vendors' claims are accurate.True or false which of the following correctly identify aspects of american cultural life in the early nineteenth century? They challenged ideas rooted in tradition and religion, such as superstitions, and inherited wealth and privilege.What does the author mean by this sentence? Edith's van can safely carry a maximum land of 920 kilograms. She wants to use her van to carry30 sacks of potatoes, each of mass 25 kilograms to the nearest kilogramand20 sacks of carots, each of mass 7. 5 kilograms to 1 decimal placeCan she definitely use her van safety in one journey?You must show your working(4 marks) Please help me as Im struggling Question 11 (1 point)(06.03 LC)What is the product of the expression, 5x(x2)? a25x2 b10x c5x3 d5x2 The enthalpy of vaporization for water is 40.7 kJ/mol. Water has a vapor pressure of 101.3 kPa at 100.0 C. Using the Clausius-Clapeyron equation, what is the vapor pressure for methanol at 73.5C? Give your answer in KPa, to the first decimal point. you are studying the movement of a positively charged substance into epithelial cells grown in culture. you determined that you can only find the substance inside the cell when atp is present. how is this particular substance getting through the membrane? the weights of bunches of bananas in the grocery store are normally distributed with a mean weight of 3.54 pounds and a standard deviation of 0.64 pounds. a random sample of four bunches is taken and the mean weight is recorded. which of the following is the mean of the sampling distribution for the mean of all possible samples of size four? a.0.89 b.1.27 c.3.54 d.5.53 The mean time to admit an emergency patient to the Mount Nittany Medical Center is 5 minutes with a standard deviation of 3 minutes. Only trauma patients are admitted to this center. Also, assume that the admission process is in fact the radiography process via an X-Ray machine.(a) What is the natural coefficient of variation for one patient?C0 : ______________(b) If the admission times of patients are independent, what will be the mean and variance of admitting a group of 50 emergency patients? What will be the coefficient of variation of a group of 50 emergency patients?t0 : ______________ 02 : ______________ C0 :______________(c) The X-Ray machine in the center may fail at any time randomly. The time to failure is exponentially distributed with a mean of 80 hours and the repair time is also exponentially distributed with a mean of 4 hours. What will be the effective mean and coefficient of variation of the admission time for a group of 50 trauma patients?te : ______________ e 2 :____________ Ce :______________(d) Determine the variability class of the squared-coefficients of variation in Parts a-c (e.g., low variability, moderate variability, or high variability.)C0 2(Part a): ______________ C0 2(Part b): ______________ Ce 2(Part c): ______________(e) In two sentences, describe how the manager of center can improve the inflated effective admission time in Part c? Jens assignment is to read at least 85 pages of a novel. Jen has read 31 pages. How many pages p does Jen have left to read? Write an inequality that represents this situation. Then solve the inequality Within a workgroup environment, if EFS keys are not backed up, you can still access EFS files when they are restored after a recovery, provided you are a member of the Administrators group.a. Trueb. False an investor is thinking about buying a $500,000 house to use as a rental property. she has $100,000 saved up for a down payment and will mortgage the remaining $400,000. the annual cash flow is projected to be $25,000. what is the potential return on investment?